Can someone help me with part a?

Can Someone Help Me With Part A?

Answers

Answer 1

Answer:

  [tex]F(x)=\left\{\begin{array}{cl}15(40-x)&\text{if $0<x<40$}\\0&\text{if $40\le x\le65$}\\15(x-65)&\text{if $x>65$}\end{array}\right.[/tex]

Step-by-step explanation:

(a) For x below 40, the number of miles per hour below 40 is (40-x). Then the fine for that speed is 15(40 -x).

For x between limits, the fine is zero.

For x above 65, the number of miles per hour above 65 is (x-65). Then the fine for that speed is 15(x -65).

  [tex]F(x)=\left\{\begin{array}{cl}15(40-x)&\text{if $0<x<40$}\\0&\text{if $40\le x\le65$}\\15(x-65)&\text{if $x>65$}\end{array}\right.[/tex]


Related Questions

Can y’all help me what do I do here please I’ll give you brainliest

Answers

Answer:

1, 2, -3, 51

Step-by-step explanation:

0 ÷ 2 = 0

0 + 1 = 1

2 ÷ 2 = 1

1 + 1 = 2

-8 ÷ 2 = -4

-4 + 1 = -3

100 ÷ 2 = 50

50 + 1 = 51

What is the equation of a line that passes through (4, 3) and has a slope of 2?

A. y=2x – 11
B. y=2x-5
C. y=2x - 1
D. y=2x - 7

Answers

i would have to say

letter C. y=2x-1

there is a number that if you multiple it by 5 first, and then add 6 to the result, you get 91. what’s the number?

Answers

Answer:

Step-by-step explanation:

Do 91-6 which equals 85. Then do 85/5 which equals 17

answer: 17

Answer:

17

Step-by-step explanation:

So first we write an equation

5x+6=91

subtract 6 from both sides

5x=85

divide by 5

so x=17

(plz give me brainliest)

Write the phrase as an expression
the quotient of 22 and a number a

Answers

Answer:

22/a

Step-by-step explanation:

The quotient of 22 and a number = 22/a

Can someone help me with #s 16 18 19 20 I WILL GIVE BRAINLIEST

Answers

Answers & Step-by-step explanation:

PEMDAS (Parenthesis, exponents, multiplication, division, addition, subtraction)

Now let's solve your problems!

___________________________________________________16.

First, we see if there are any parentheses. Since there are no parentheses, we will evaluate the exponent 3² first.

After this, our problem now looks like this:

[tex]\frac{41 - 9}{6 * 3 -26}[/tex]

Next, since multiplication (not subtraction) is next on the list, we will evaluate 6 × 3, which equals 18.

After that, we will subtract:

41 - 9 = 32

18 - 26 = -8

We now have 32/-8 or 4/-1 or simply -4.

The answer is: -4___________________________________________________17.

With this, one, note the parentheses. This means that we will do these operations first.

(21 - 4²) = 5

(3 + 1) = 4

The problem now looks like this:

[tex]\frac{5 * 4}{8^{2}-44}[/tex]

Next, we do the exponents:

8² = 64

After that, we do the multiplication of 5 x 4, which equals 20.

We now have 20/64 - 44, which simplifies to 20/20 or 1/1 or 1.

The answer is: 1___________________________________________________18.

Again, we see parentheses, so that means we need to do these operations first.

(6² - 20) = 16

We now have:

48 ÷ 16 + 5

48/16 = 3 and 3 + 5 = 8

The answer is: 8___________________________________________________19.

Here, we see an exponent 3³, so we'll need to evaluate this first.

3³ = 27

We now have:

4 × 27 ÷ 18 + 14

4 × 27 = 108

108/18 = 6

And finally:

6 + 14 = 20

The answer is: 20___________________________________________________20.

Here we again see the parentheses, which means we need to evaluate the subtraction first.

3 - 8 = -5

We now have:

11 + 2 (-5)

2 × -5 = -10

11 + -10 (same as 11 - 10) = 1

The answer is: 1___________________________________________________I AM ALWAYS HAPPY TO HELP :)

Comelia and Christopher are arguing about Real Number (R) Sets. Comelia says that all Whole (W) numbers are Rational (Q) and Christopher
says that all Rational (Q) numbers are Whole (W). Who is correct and explain why?
Unread
Search entries or author
EL
1
Edit Insert Format Tools
Table
<
>
2
Paragraph
B.
12pt
I U Avert²v
I

Answers

Answer: Comelia is correct

====================================================

Explanation:

We're told that "Christopher  says that all Rational (Q) numbers are Whole (W)", which makes Christopher not correct. Some rational numbers are whole numbers. For instance, the number 7 = 7/1 is rational and it's a whole number as well.

However something like 1/2 is rational, but it's not a whole number. A whole number doesn't have any fractional or decimal part to it. It can be thought of the number of something.

Comelia is correct because all whole numbers are rational. If x is some whole number, then x = x/1 is rational as well. Replace x with any whole number you want. Her statement does not work in reverse as shown above.

When drawing a Venn diagram, the circle for "whole numbers" will be entirely inside the circle for "rational numbers", and not the other way around.


Find the arc length of the semicircle.
Either enter an exact answer in terms of it or use 3.14 for IT and enter your answer as a decimal.

Answers

A semicircle is a half of a circle.

Find the circumference of a full circle then divide in half.

Circumference = pi x r x 2

Circumference = 3.14 x 9 x 2 = 56.52

Semicircle = 56.52/2 = 28.26

5 + 2x < 2x + 6
what is x?

Answers

Answer:

x = infinite amount of solutions

Step-by-step explanation:

Step 1: Write out inequality

5 + 2x < 2x + 6

Step 2: Solve

5 < 6

Since we can see that 6 is indeed bigger than 5, it makes the inequality true. If you plug in any number x, it will still be true. Therefore, we have an infinite amount of solutions,

What is a Metric prefix?

Answers

Answer:

   

Step-by-step explanation:

A metric prefix is a unit prefix that precedes a basic unit of measure to indicate a multiple or submultiple of the unit. All metric prefixes used today are decadic. Each prefix has a unique symbol that is prepended to any unit symbol.

This is the definition

PLEASE HELP! 15 POINTS Order the expressions from greatest to least coefficient of x. Place the expression with the greatest coefficient of x at the top. 12+4x, 13x, 15-x, 14-7x----------My answer: 13x, 14-7x, 12+4x, 15-x

Answers

Answer:

you're right; 13x, 14 - 7x, 12 + 4x, 15 - x

13, 7, 4, 1

What is true about the number 2.586? Check all that apply. The 8 is in the hundredths place. The 6 is in the thousandths place. The 5 is in the ones place. This number is read as "two and five hundred eighty-six thousandths." 2.586 is equivalent to (2 × 1) + (5 × 0.1) + (8 × 0.01) + (6 × 0.001).

Answers

Answer:

The 8 is in the hundredths place.

The 6 is in the thousandths place.

2.586 is equivalent to (2 × 1) + (5 × 0.1) + (8 × 0.01) + (6 × 0.001).

Answer:

Step-by-step explanation: A  B  E   are the right ones,     credit to apo16

g ( h + 3/5 ) = 11 solve for h

Answers

Answer:

h = 11/g - 3/5

Step-by-step explanation:

[tex]g ( h + \frac{3}{5} ) = 11 \\\\Divide\:both\:sides\:of\:the\:equation\:by\:g \:;\\\frac{g(h+\frac{3}{5}) }{g} = \frac{11}{g} \\\\h + \frac{3}{5} = \frac{11}{g} \\\\Move\: 3/5\: to\: the\: right\:and\:change\:its\:sign\\h = \frac{11}{g} -\frac{3}{5}[/tex]

1.11 People fit comfortably in a 5 feet by 5 feet area. Use this value to estimate the size of a crowd that is 5 feet deep on BOTH sides of the street along a 5-mile section of a parade route.
(4 Points)

Answers

Answer:

in 5ft by 5ft = 25ft^2, there can be 1 person

A normal street is 30ft wide

Then, if we want the width such that is 5 ft deep (from both sides)

Then we should subtract two times 5 ft:

30ft - 5ft - 5ft = 20ft.

Now the length is 5 miles

and 1 mile = 5280 ft

then 5 miles = 5*( 5280 ft) = 26,400 ft.

Then the area is:

20ft by 26,400 ft. =528,800  ft^2

And we know that in 25 ft^2 we can fit one person.

How many spaces of 25ft^2 we have in the 528,800 ft^2?

this is:

N =  528,800 ft^2/25ft^2 = 21,152

This means that in the street 21,152 people can fit comfortably.

The size of the crowd is  21,152 people.

Phil weighs 120 pounds and is gaining ten pounds each month. Phil weighs 150 pounds and is gaining 4 pounds each month. How many months, m, will it take for Bill to weigh the same as Phil?

Answers

Answer:

5 months

Step-by-step explanation:

I'm assuming there was a typo and said that Phil weighed both amounts so I'm assuming that the second amount is Bill's weight

We can use LCM for this

So lets make a list for both of them

Phil:120, 130, 140, 150, 160, 170, 180, 190, 200

Bill: 150, 154, 158, 162, 166, 170, 174

As you can see here, after 5 months, they will have the same weight.

We can check our work to.

lets do 120+(10*5)=170

and 150+(4*5)=170

I hope this makes sense

Answer:

5 months

Step-by-step explanation:

lets chart it like i did as you can see they both started at 120 and 150 then 5 month later there both 170

bill              phil

start . 120      150

1m.130       154

2m.140       158

3m.150       162

4m.160       166

5m.170        170

Which is a common denominator of 1/5 and 1/10

Answers

Answer:

1

Step-by-step explanation:

i'm in middle school answering a question that can be solved

Answer:

10

Step-by-step explanation:

take LCM of 5 and 10 to find the common denominator

win) = 2n + 1; Find w(6)

Answers

Answer:

13

Step-by-step explanation:

w(n) = 2n + 1;

Let n = 6

w(6)= 2*6 +1

      = 12 +1

      = 13

Answer:

[tex]\Huge \boxed{w(6)=13}[/tex]

[tex]\rule[225]{225}{2}[/tex]

Step-by-step explanation:

[tex]w(n)=2n+1[/tex]

Replace n with 6.

[tex]w(6)=2(6)+1[/tex]

Evaluating:

[tex]w(6)=12+1[/tex]

[tex]w(6)=13[/tex]

[tex]\rule[225]{225}{2}[/tex]

i would love if you could answer very soon

Which ordered pair is a solution to the system of inequalities?
S y> -2
x + y < 4

Answers

Answer:

4

Step-by-step explanation: you have t do y plus 10 and then add 4  minus -2

Is y=x^2-6 a linear function and why

Answers

Answer:

No.

Step-by-step explanation:

No, y=x²-6 is not a linear function.

This is because all linear functions have the highest exponent of 1. Anything equation that has a variable raised to a power other than 1 is not linear.

The equation provided as the highest exponent of 2. Thus, it's not linear.

From the provided graph, we can also see that this is quadratic instead of linear. The graph is a parabola instead of a line.

Answer:

NO! y=x^2-6 is nonlinear

Step-by-step explanation:

A great teacher once told me the easiest way to tell if an equation is linear or not, by just looking at it is, if you see that the equation contains an exponent then you should automatically know that it is nonlinear. Therefore the equation y=x^2-6 is nonlinear!!!

Which represents a function? y x y -3 -5 10 {(-8, -2), (-4,1). (0,-2), (2, 3), (4,-4)} -2 -3 5 {(-12,4),(-6, 10), (-4, 15).(-8, 18). (-12, 24) -2 -3 4 0 -1 0 0 0 0 5 -10 1 -1​

Answers

The 2nd one , because your x value should only repeat one time , and it follows those standards.

The tabular data shown in the image attached does not represent a function.

What is function?

A function is a relation between a dependent and independent variable. We can write the examples of function as -

y = f(x) = ax + b

y = f(x, y, z) = ax + by + cz

Given is to find which of the given tables represent a function.

For a table to represent a function, for every unique value of {x}, there should be only one possible value of {y}. This means that if for a function f(x), at {x} = 1, there exists two or more values of {y}, then it is not a function. From the image, it can be seen that the relations given in tables does not represent a function. This is because -

Table {1} -

for {x} = - 3, there exists two values of y

Table {3} -

for {x} = 0, there exists two values of y

Therefore, the tabular data shown in the image attached does not represent a function.

To solve more questions on functions, visit the link below-

https://brainly.com/question/30194223

#SPJ7

square root of 2025 by prime factorisation method​
Please explain step by step

Answers

Answer:

45

Step-by-step explanation:2025....use the smallest prime number to divide the number 2025 till you get to a point were it can't be divided again . Group each prime number with a partner of its value like this :

5×5×3×3×3×3........ then use bracket to separate them like this (5×5)(3×3)(3×3)........ then take a number from each 5×3×3 then multiply to get 45

I am in class I really need help the teacher is gonna call on me help please

Answers

Answer:

5/10 or 1/2

Step-by-step explanation:

5-4 is 1, and 1 squared is still 1.

the cube root of 27 is 3

7 to the 0 power is 1

1 + 3 + 1 is 5

5 over 10 is 1/2

Answer:

1/2

Step-by-step explanation:

(5-4)²+∛27+[tex]7^{0}[/tex]

1²+∛27+[tex]7^{0}[/tex]

1+∛27+[tex]7^{0}[/tex]

1+3+[tex]7^{0}[/tex]

1+3+1

4+1

5/10=1/2

Add the polynomials (7x3−2x2−12)+(−3x3−8x2+10x)

Answers

Answer:

See below

Step-by-step explanation:

● (7x^3 - 2x^2 -12) + (-3x^3 - 8x^2 +10x)

● 7x^3 - 2x^2 - 12- 3x^3 - 8x^2 + 10x

Combine like terms

● 7x^3 - 3x^3 -2x^2 - 8x^2 -12 + 10x

● 4x^3 - 10x^3 +10x -12

Answer:

Addition:::::4x^3-10x^2+10x-12

Solve -17 + n over 5 = 33 please help :(

Answers

the answer is n=182 !
Omg I was stuck on that too thank you so much

What’s the value of x

Answers

Answer:

2

Step-by-step explanation:

20-7x=6x-6

20+6=6x+7x

26=13x

x=2

What are the two solutions to the absolute value equation |2x+3|=7

Answers

Step-by-step explanation: We can solve the equation |2x + 3| = 7 by

first splitting things up into two separate equations.

The first equation will look just like the original problem,

minus the absolute value signs and we have 2x + 3 = 7.

The second equation will look just like the first, only the right

side will be changed to a negative and we have 2x + 3 = -7.

Now solve each equation from here.

In our first equation, subtract 3 from both sides to get 2x = 4.

Then divide both sides by 2 and x = 2.

In our second equation, subtract 2 from both sides to get 2x = -10.

Now, divide both sides by 2 and we have x = -5.

So either x = 2 or x = -5.

1) How would you solve consecutive integers? 2) What happen to the equations when you solve consecutive EVEN integers? 3) What about consecutive ODD integers? ​

Answers

Answer:

(a) make use of average values; (b) use a variable (n) and consecutive values: n+1, n+2, and so onEven integers differ by 2 instead of 1, so they would be represented by n, n+2, n+4, and so on.Odd integers also differ by 2, so they would have the same representation.

Step-by-step explanation:

1) If the sum of consecutive integers is given, I usually solve the problem by considering the average of those integers: the sum divided by the number of integers. (See https://brainly.com/question/17307802 for an example.)

If some other relation is given then quite often a variable is assigned to one of the integers, often the smallest. If they are consecutive, then the others may be n+1, n+2, n+3 and so on.

__

2) If they are consecutive even or odd, then they differ by 2 instead of 1: n, n+2, n+4, and so on.

__

3) That difference of 2 is the same whether the integers are consecutive even or consecutive odd. The representation used does not change.

_____

Additional comment

In some instances, you need to ensure that the integers are even or odd, so you might use a representation of 2n for an even integer and 2n+1 for an odd integer. This is rarely an issue in consecutive integer problems.

Find the smallest number which when divided by 12,15,18&27 leaves as remainder 8,11,14,23 respectively.

Answers

Answer:

[tex] \boxed{ \bold{ \huge{ \boxed{ \sf{536}}}}}[/tex]

Step-by-step explanation:

Solution :

Here, 12 - 8 = 4

15 - 11 = 4

18 - 14 = 4

27 - 23 = 4

Thus, every divisor is greater than its remainder by 4. So, the required smallest number is the difference of the L.C.M of the given number and 4

Finding the L.C.M

First of find the prime factors of each numbers

12 = 2 × 2 × 3

15 = 3 × 5

18 = 3 × 3 × 3

27 = 3 × 3 × 3

Take out the common prime factors : 3 , 3 and 3

Also take out the other remaining prime factors : 2 , 2 and 5

Now, Multiply those all prime factors and obtain L.C.M

L.C.M = Common factors × Remaining factors

= 3 × 3 × 3 × 2 × 2 × 5

= 540

L.C.M of 12 , 15 , 18 and 27 = 540

So, The required smallest number = 540 - 4

= 536

Hope I helped!

Best regards!!

Determine the domain of the function (fog)(x) where f(x)=3x-1/x-4 and g(x)=x+1/x

Answers

Answer: (-∞,-1) ∪ (0,+∞)

Step-by-step explanation: The representation fog(x) is a representation of composite function, meaning one depends on the other.

In this case, fog(x) means:

fog(x) = f(g(x))

fog(x) = [tex]3(x+\frac{1}{x} )-\frac{1}{x+\frac{1}{x} } -4[/tex]

[tex]fog(x)=3x+\frac{3}{x} -\frac{1}{\frac{x^{2}+x}{x} } -4[/tex]

[tex]fog(x)=3x+\frac{3}{x} -\frac{x}{x^{2}+x} -4[/tex]

[tex]fog(x)=\frac{3x^{2}(x^{2}+x)+3(x^{2}+x)-x-4x(x^{2}+x)}{x(x^{2}+x)}[/tex]

[tex]fog(x)=\frac{3x^{4}+3x^{3}+3x^{2}+3x-x-4x^{3}+4x^{2}}{x(x^{2}+x)}[/tex]

[tex]fog(x)=\frac{3x^{4}-x^{3}-x^{2}+2x}{x(x^{2}+x)}[/tex]

This is the function fog(x).

The domain of a function is all the values the independent variable can assume.

For fog(x), denominator can be zero, so:

[tex]x(x^{2}+x) \neq 0[/tex]

If x = 0, the function doesn't exist.

[tex]x^{2}+x \neq0[/tex]

[tex]x(x+1) \neq0[/tex]

[tex]x+1\neq0[/tex]

[tex]x\neq-1[/tex]

Therefore, the domain of this function is: -∞ < -1 or x > 0

How can the additive inverse be used to evaluate the problem below?

-3-(-6) ???

Answers

Answer:

Hey there!

It can be used because -3-(-6) is equal to -3+6, which is 3.

Let me know if this helps :)

3
Exress 10° in standard form
1
10
100

Answers

Answer:

1

Step-by-step explanation:

In general [tex]a^{0}[/tex] = 1 ( where a is a real number ), thus

[tex]10^{0}[/tex] = 1